Brilliant Summation Contest Season-1 (Part 2)

Hi Brilliant! This is the sequel of the first part of Brilliant Summation Contest Season-1.

Update: This contest has been ended. Thanks for everyone's participation.

The aims of the Summation contest are to improve skills in the computation of sums, to learn from each other as much as possible, and of course to have fun. Anyone here may participate in this contest.

The rules are as follows:

  • I will start by posting the first problem. If there is a user solves it, then they must post a new one.

  • You may only post a solution of the problem below the thread of problem and post your proposed problem in a new thread. Put them separately.

  • Only make substantial comment that will contribute to the discussion.

  • Make sure you know how to solve your own problem before posting it in case there is no one can answer it within 48 hours, then you must post the solution and you have a right to post another problem.

  • If the one who solves the last problem does not post his/her own problem after solving it within a day, then the one who has a right to post a problem is the last solver before him/her.

  • You are NOT allowed to post a multiple summation problem.

  • Problems must be purely of summation. They shouldn't have integrals and products in them. Solutions can follow methods that use integrals and products.

  • It is NOT compulsory to post original problems. But make sure it has not been posted on brilliant.

  • There is no restriction in the standard of summations.

Please post your solution and your proposed problem in a single new thread.

Format your post is as follows:

1
2
3
4
5
6
7
**SOLUTION OF PROBLEM xxx (number of problem) :**

**[Post your solution here]**

**PROBLEM xxx (number of problem) :**

**[Post your problem here]**

The comments will be easiest to follow if you sort by "Newest":

#Calculus

Note by Aditya Kumar
4 years, 11 months ago

No vote yet
1 vote

  Easy Math Editor

This discussion board is a place to discuss our Daily Challenges and the math and science related to those challenges. Explanations are more than just a solution — they should explain the steps and thinking strategies that you used to obtain the solution. Comments should further the discussion of math and science.

When posting on Brilliant:

  • Use the emojis to react to an explanation, whether you're congratulating a job well done , or just really confused .
  • Ask specific questions about the challenge or the steps in somebody's explanation. Well-posed questions can add a lot to the discussion, but posting "I don't understand!" doesn't help anyone.
  • Try to contribute something new to the discussion, whether it is an extension, generalization or other idea related to the challenge.
  • Stay on topic — we're all here to learn more about math and science, not to hear about your favorite get-rich-quick scheme or current world events.

MarkdownAppears as
*italics* or _italics_ italics
**bold** or __bold__ bold

- bulleted
- list

  • bulleted
  • list

1. numbered
2. list

  1. numbered
  2. list
Note: you must add a full line of space before and after lists for them to show up correctly
paragraph 1

paragraph 2

paragraph 1

paragraph 2

[example link](https://brilliant.org)example link
> This is a quote
This is a quote
    # I indented these lines
    # 4 spaces, and now they show
    # up as a code block.

    print "hello world"
# I indented these lines
# 4 spaces, and now they show
# up as a code block.

print "hello world"
MathAppears as
Remember to wrap math in \( ... \) or \[ ... \] to ensure proper formatting.
2 \times 3 2×3 2 \times 3
2^{34} 234 2^{34}
a_{i-1} ai1 a_{i-1}
\frac{2}{3} 23 \frac{2}{3}
\sqrt{2} 2 \sqrt{2}
\sum_{i=1}^3 i=13 \sum_{i=1}^3
\sin \theta sinθ \sin \theta
\boxed{123} 123 \boxed{123}

Comments

Problem 27:

Prove:

n=(eπ)n2=(π2)14Γ(34)\sum _{ n=-\infty }^{ \infty }{ { \left( -e^{-\pi} \right) }^{ { n }^{ 2 } } } =\frac { { \left( \frac { \pi }{ 2 } \right) }^{ \frac { 1 }{ 4 } } }{ \Gamma \left( \frac { 3 }{ 4 } \right) }

Aditya Kumar - 4 years, 11 months ago

Log in to reply

We have a proof on brilliant

n=(eπ)n2=(π)14Γ(34)\sum _{ n=-\infty }^{ \infty }{ { \left( e^{-\pi} \right) }^{ { n }^{ 2 } } } =\frac { { \left( \pi \right) }^{ \frac { 1 }{ 4 } } }{ \Gamma \left( \frac { 3 }{ 4 } \right) }

Now use this formula ϕ(e2π/n)=(π)14Γ(34)n1/4r2,2/n221/4r2,2\phi(-e^{-2\pi/n})=\frac { { \left( \pi \right) }^{ \frac { 1 }{ 4 } } }{ \Gamma \left( \frac { 3 }{ 4 } \right) } \frac{n^{1/4}r_{2,2/n^2}}{2^{1/4}r_{2,2}}

rk,nr_{k,n} is a function of DedeKind Eta function. Which have some formulaes to solve.

r2,2=21/8andr2,1/2=21/8r_{2,2}=2^{1/8} \text{and} r_{2,1/2}=2^{-1/8}

Thus , you get (π2)1/4Γ(3/4)\frac{(\frac{\pi}{2})^{1/4}}{\Gamma(3/4)}

Aman Rajput - 4 years, 11 months ago

Log in to reply

At least could you elaborate on step 2?

Aditya Kumar - 4 years, 11 months ago

Log in to reply

@Aditya Kumar I used this identity from a ramanujan paper

Aman Rajput - 4 years, 11 months ago

This is evaluated in ramanujan's notebook which I have but there's no proof for the formula used :/ . So I was trying to prove the above q-product

Aditya Narayan Sharma - 4 years, 11 months ago

Log in to reply

@Aditya Narayan Sharma share your q product .... if i have time i can solve it easily

Aman Rajput - 4 years, 11 months ago

Log in to reply

@Aman Rajput No, I m not talking bout the q-product actually. I m talking about the above formula , he used q-products and generalisations to come to this. So share if you have a proof of this , coz I didnt find

Aditya Narayan Sharma - 4 years, 11 months ago

Problem 17 :

Prove That

n=02n1n!r=0n(4r+1)=π+2log(1+2)42 \displaystyle \sum_{n=0}^{\infty} \dfrac{2^{n-1} n!}{\prod_{r=0}^{n} (4r+1)} = \dfrac{\pi + 2\log(1+\sqrt{2})}{4 \sqrt{2}}

This problem has been solved by Aditya Kumar.

Ishan Singh - 4 years, 11 months ago

Log in to reply

Solution to Problem 17:

S=n=02n1n!r=0n(4r+1)S=\sum _{ n=0 }^{ \infty } \frac { 2^{ n-1 }n! }{ \prod _{ r=0 }^{ n } (4r+1) }

S=n=0[2n1Γ(n+1)Γ(54)Γ(n+54)]S=\sum _{ n=0 }^{ \infty } \left[ \frac { { 2 }^{ -n-1 }\Gamma \left( n+1 \right) \Gamma \left( \frac { 5 }{ 4 } \right) }{ \Gamma \left( n+\frac { 5 }{ 4 } \right) } \right]

Using beta function, we get:

S=n=0[2n1B(n+1,54).(n+54)]S=\sum _{ n=0 }^{ \infty } \left[ { 2 }^{ -n-1 }B\left( n+1,\frac { 5 }{ 4 } \right) .\left( n+\frac { 5 }{ 4 } \right) \right]

S=12n=0[2nB(n+1,54).n]+58n=0[2nB(n+1,54)]S=\frac { 1 }{ 2 } \sum _{ n=0 }^{ \infty } \left[ { 2 }^{ -n }B\left( n+1,\frac { 5 }{ 4 } \right) .n \right] +\frac { 5 }{ 8 } \sum _{ n=0 }^{ \infty } \left[ { 2 }^{ -n }B\left( n+1,\frac { 5 }{ 4 } \right) \right]

S=1201{n=0[2n.n.tn(1t)14]dt}+5801{n=0[2n.tn(1t)14]dt}S=\frac { 1 }{ 2 } \int _{ 0 }^{ 1 }{ \left\{ \sum _{ n=0 }^{ \infty }{ \left[ { 2 }^{ -n }.n.{ t }^{ n }{ \left( 1-t \right) }^{ \frac { 1 }{ 4 } } \right] } dt \right\} } +\frac { 5 }{ 8 } \int _{ 0 }^{ 1 }{ \left\{ \sum _{ n=0 }^{ \infty }{ \left[ { 2 }^{ -n }.{ t }^{ n }{ \left( 1-t \right) }^{ \frac { 1 }{ 4 } } \right] } dt \right\} }

Now I'll use the following geometric progression summations:

A=n=0[(xt)n(1t)14]=(1t)141xt,B=n=0[(xt)n.n.(1t)14]=xt(1t)14(1xt)2A=\sum _{ n=0 }^{ \infty }{ \left[ { \left( xt \right) }^{ n }{ \left( 1-t \right) }^{ \frac { 1 }{ 4 } } \right] } =\frac { { \left( 1-t \right) }^{ \frac { 1 }{ 4 } } }{ 1-xt } ,\quad B=\sum _{ n=0 }^{ \infty }{ \left[ { \left( xt \right) }^{ n }{ .n.\left( 1-t \right) }^{ \frac { 1 }{ 4 } } \right] } =\frac { { xt\left( 1-t \right) }^{ \frac { 1 }{ 4 } } }{ { \left( 1-xt \right) }^{ 2 } }

S=5401(1t)142tdt+01x(1x)14(2x)2dxS=\frac { 5 }{ 4 } \int _{ 0 }^{ 1 }{ \frac { { \left( 1-t \right) }^{ \frac { 1 }{ 4 } } }{ 2-t } dt } +\int _{ 0 }^{ 1 }{ \frac { x{ \left( 1-x \right) }^{ \frac { 1 }{ 4 } } }{ { \left( 2-x \right) }^{ 2 } } dx }

The first integral can be easily evaluated. The second one is also easily evaluated. I was screwed up and I couldn't evaluate it. So I asked for it here.

Hence the final answer on doing some simplification is:

n=02n1n!r=0n(4r+1)=π+2log(1+2)42\boxed{\displaystyle\sum_{n=0}^{\infty} \dfrac{2^{n-1} n!}{\prod_{r=0}^{n} (4r+1)} = \dfrac{\pi + 2\log(1+\sqrt{2})}{4 \sqrt{2}}}

Aditya Kumar - 4 years, 11 months ago

Problem 18:

Prove that:

k=0n(n+knk)(1)n+k2k+1=22n+1cos(2(n1)π3)  ?\sum_{k=0}^n \binom{n+k}{n-k}\frac{(-1)^{n+k}}{2k+1}=-\frac{2}{2n+1}\,\cos\left(\frac{2(n-1)\pi}{3}\right)\;\text{?}

This Problem has been solved by Ishan Singh.

Aditya Kumar - 4 years, 11 months ago

Log in to reply

Solution Of Problem 18 :

Proposition : Sn=r=0n(1)r(2n+1rr)=23sin(2(n+1)π3) \text{S}_{n} = \sum_{r=0}^{n} (-1)^r \dbinom{2n+1-r}{r} = \dfrac{2}{\sqrt{3}} \sin \left( \dfrac{2 (n+1) \pi}{3} \right)

Proof : Since (nr)=0 \dbinom{n}{r} = 0 for r>n r > n , we can rewrite the sum as

Sn=r=0(1)r(2n+1rr) \text{S}_{n} = \sum_{r=0}^{\infty} (-1)^r \dbinom{2n+1-r}{r}

From the Binomial Theorem, we see that the sum is the coefficient of xnx^n in

f(x)=xn(1x)2n+1+xn1(1x)2n+f(x) = x^n (1-x)^{2n+1} + x^{n-1} (1-x)^{2n} + \ldots

=(1x)n+1x2x+1 = \dfrac{(1-x)^{n+1}}{x^2-x+1}

Note that,

n=0Ur(a)xr=1x22ax+1 \sum_{n=0}^{\infty}{{U}_{r}(a) {x}^{r}} = \dfrac{1}{x^2 -2ax+1}

where Ur(x) U_{r} (x) is the Chebyshev Polynomial of the second kind.

Putting a=12a = \dfrac{1}{2}, we see that,

f(x)=(1x)n+1n=0Ur(12)xr() f(x) = (1-x)^{n+1} \sum_{n=0}^{\infty}{{U}_{r} \left( \dfrac{1}{2} \right) {x}^{r}} \quad (*)

To calculate the coefficient of xnx^n in () (*) , we see that coefficient of xnrx^{n-r} for fixed rr in (1x)n+1(1-x)^{n+1} is (1)nr(n+1nr) (-1)^{n-r} \dbinom{n+1}{n-r} , so coefficient of xnx^n is,

r=0n(1)nr(n+1nr)Ur(12) \sum_{r=0}^{n} (-1)^{n-r} \dbinom{n+1}{n-r} U_{r} \left(\dfrac{1}{2}\right)

Substituting rr1 r \mapsto r-1 , we have,

Sn=r=1n+1(1)n+1r(n+1n+1r)Ur1(12) \text{S}_{n} = \sum_{r=1}^{n+1} (-1)^{n+1-r} \dbinom{n+1}{n+1-r} U_{r-1} \left(\dfrac{1}{2}\right)

=(1)n+1r=1n+1(1)r(n+1r)Ur1(12)((nnr)=(nr)) = (-1)^{n+1} \sum_{r=1}^{n+1} (-1)^{r} \dbinom{n+1}{r} U_{r-1} \left(\dfrac{1}{2}\right) \quad \quad \left( \because \dbinom{n}{n-r} = \dbinom{n}{r} \right)

=(1)n+1(23)r=1n+1(1)r(n+1r)sin(rπ3)(Ur1(12)=sin(rπ3)) = (-1)^{n+1} \left( \dfrac{2}{\sqrt{3}} \right) \sum_{r=1}^{n+1} (-1)^{r} \dbinom{n+1}{r} \sin \left( \dfrac{r \pi}{3} \right) \quad \quad \left( \because U_{r-1} \left(\dfrac{1}{2}\right) = \sin \left( \dfrac{r \pi}{3} \right) \right)

(1)n+1(23)r=0n+1(1)r(n+1r)sin(rπ3)(sin0=0) (-1)^{n+1} \left( \dfrac{2}{\sqrt{3}} \right) \sum_{r=0}^{n+1} (-1)^{r} \dbinom{n+1}{r} \sin \left( \dfrac{r \pi}{3} \right) \quad \quad \left( \because \sin 0 = 0 \right)

Now, r=0n+1(1)r(n+1r)sin(rπ3)=(r=0n+1(1)r(n+1r)eirπ3)=(1)n+1sin(2(n+1)π3)\displaystyle \sum_{r=0}^{n+1} (-1)^{r} \dbinom{n+1}{r} \sin \left( \dfrac{r \pi}{3} \right) = \Im \left( \sum_{r=0}^{n+1} (-1)^{r} \dbinom{n+1}{r} e^{\frac{i r \pi}{3}} \right) = (-1)^{n+1} \sin \left( \dfrac{2 (n+1) \pi}{3} \right)

    Sn=23sin(2(n+1)π3) \implies \text{S}_{n} = \dfrac{2}{\sqrt{3}} \sin \left( \dfrac{2 (n+1) \pi}{3} \right) \quad \square

Corollary : Sn=k=0n(1)n+k(n+1+k2k+1)=23sin(2(n+1)π3)\text{S}_{n} = \sum_{k=0}^n (-1)^{n+k} \dbinom{n+1+k}{2k+1} = \dfrac{2}{\sqrt{3}} \sin \left( \dfrac{2 (n+1) \pi}{3} \right)

Proof : k=0n(1)n+k(n+1+k2k+1)=k=0n(1)n+k(n+1+knk)=k=0n(1)k(2n+1kk)=Sn\sum_{k=0}^n(-1)^{n+k} \dbinom{n+1+k}{2k+1} = \sum_{k=0}^n(-1)^{n+k} \dbinom{n+1+k}{n-k} = \sum_{k=0}^n(-1)^k \dbinom{2n+1-k}{k} = \text{S}_{n}

((nr)=(nnr) & r=0nf(k)=r=0nf(nk))\left( \because \dbinom{n}{r} = \dbinom{n}{n-r} \ \text{\&} \ \sum_{r=0}^{n} f(k) = \sum_{r=0}^{n} f(n-k) \right) \quad \square

Now,

λ=k=0n(1)n+k2k+1(n+knk) \lambda = \sum_{k=0}^n\dfrac{(-1)^{n+k}}{2k+1} \dbinom{n+k}{n-k}

=k=0n(1)n+k(n+k)!(2k+1)!(nk)! = \sum_{k=0}^n \dfrac{(-1)^{n+k}(n+k)!}{(2k+1)!(n-k)!}

=12n+1k=0n(1)n+k(n+k)![(n+k+1)+(nk)](2k+1)!(nk)! =\dfrac{1}{2n+1}\sum_{k=0}^n\dfrac{(-1)^{n+k}(n+k)!\left[(n+k+1)+(n-k)\right]}{(2k+1)!(n-k)!}

=12n+1k=0n((1)n+k(n+1+k2k+1)(1)n1+k(n+k2k+1)) =\dfrac{1}{2n+1}\sum_{k=0}^n\left((-1)^{n+k} \dbinom{n+1+k}{2k+1} - (-1)^{n-1+k} \dbinom{n+k}{2k+1} \right)

=12n+1(SnSn1) =\dfrac{1}{2n+1} \left(\text{S}_{n} - \text{S}_{n-1}\right)

Using the Proposition and simplifying, we have,

λ=22n+1cos(2(n1)π3) \lambda = -\dfrac{2}{2n+1} \, \cos\left(\frac{2(n-1)\pi}{3}\right) \quad \square


Note : Coefficient of xnx^n in the Proposition can also be found by splitting 1x2x+1\dfrac{1}{x^2-x+1} as partial fraction and expanding using Infinite GP.

Ishan Singh - 4 years, 11 months ago

Problem 22:

Prove the identity:

k=1nHknk+1=(Hn+1)2Hn+1(2)\sum _{ k=1 }^{ n }{ \frac { { H }_{ k } }{ n-k+1 } } ={ \left( { H }_{ n+1 } \right) }^{ 2 }-{ H }_{ n+1 }^{ \left( 2 \right) }

This problem has been solved by Aditya Sharma, Mark Hennings and Ishan Singh.

Aditya Kumar - 4 years, 11 months ago

Log in to reply

Let S(r)=k=0nHrkk\displaystyle S(r)=\sum_{k=0}^{n} \frac{H_{r-k}}{k} for r>0r>0 with S(0)=0S(0)=0.

We have , S(n+1)=k=1nHknk+1=k=1nHnk+1k=k=1nHnkk+2Hnn+1=S(n)+2Hnn+1\displaystyle S(n+1) = \sum_{k=1}^{n} \frac{H_k}{n-k+1} = \sum_{k=1}^{n}\frac{H_{n-k+1}}{k} = \sum_{k=1}^{n}\frac{H_{n-k}}{k} + \frac{2H_n}{n+1} = S(n)+\frac{2H_n}{n+1}

Telescoping above we have , S(n+1)=k=0n2Hkk+1\displaystyle S(n+1) = \sum_{k=0}^{n}\frac{2H_k}{k+1}

Lemma :k=0n[(k+1)pkp]Hk(m)=(n+1)pHn(m)Hn(mp)\displaystyle \text{Lemma :} \large \boxed{\sum_{k=0}^{n} [(k+1)^p-k^p]H_{k}^{(m)} = (n+1)^p H_{n}^{(m)}-H_{n}^{(m-p)}}

Proof :\text{Proof :} Using summation by parts we have ,

k=0n[(k+1)pkp]Hk(m)=(n+1)pHn+1(p)k=0n(k+1)(mp)=(n+1)pHn+1(p)Hn+1(mp)=(n+1)Hn(p)Hn(mp)\displaystyle \sum_{k=0}^{n} [(k+1)^p-k^p]H_{k}^{(m)} = (n+1)^p H_{n+1}^{(p)} - \sum_{k=0}^{n} (k+1)^{(m-p)} = (n+1)^p H_{n+1}^{(p)} - H_{n+1}^{(m-p)} = (n+1)H_{n}^{(p)} - H_{n}^{(m-p)} & thus proved.

We'll use a well known Harmonic sum identity , k=1nHkk=12((Hn)2+Hn(2))\displaystyle \sum_{k=1}^{n} \frac{H_k}{k} = \frac{1}{2}((H_n)^2+H_{n}^{(2)})

Applying the lemma for p=1,m=1p=-1,m=1 we have,

S(n+1)=2k=0nHkk+1=k=1n2Hkk+Hnn+1Hn(2)\displaystyle S(n+1) = 2\sum_{k=0}^{n} \frac{H_k}{k+1} = \sum_{k=1}^{n} \frac{2H_k}{k} + \frac{H_n}{n+1}-H_{n}^{(2)}

Simplifying we get , k=1nHknk+1=(Hn+1)2Hn+1(2)\displaystyle \sum_{k=1}^{n} \frac{H_k}{n-k+1} = (H_{n+1})^2-H_{n+1}^{(2)}

Aditya Narayan Sharma - 4 years, 11 months ago

Log in to reply

The generating function of the S(n)S(n) is n=1S(n)xn+1=n=1Hnxn×n=11nxn=ln2(1x)1x \sum_{n=1}^\infty S(n) x^{n+1} = \sum_{n=1}^\infty H_n x^n \times \sum_{n=1}^\infty \tfrac{1}{n} x^n = \frac{\ln^2(1-x)}{1-x} so the result follows using the known formulae for the generating functions for Hn(2)H_n^{(2)} and for (Hn)2(H_n)^2.

Mark Hennings - 4 years, 11 months ago

We have,

S=k=1nHknk+1 \text{S} = \sum_{k=1}^{n}\dfrac{H_{k}}{n-k+1}

=k=1nr=1k1r(nk+1) = \sum_{k=1}^{n} \sum_{r=1}^{k} \dfrac{1}{r(n-k+1)}

=r=1nk=rn1r(nk+1) = \sum_{r=1}^{n} \sum_{k=r}^{n} \dfrac{1}{r(n-k+1)}

=r=1nk=rn1r(kr+1) = \sum_{r=1}^{n} \sum_{k=r}^{n} \dfrac{1}{r(k-r+1)}

=r=1nk=rn(kr+1+r)r(kr+1)(k+1) = \sum_{r=1}^{n} \sum_{k=r}^{n} \dfrac{(k-r+1 + r)}{r(k-r+1)(k+1)}

=r=1nk=rn1(k+1)[1(kr+1)+1r] = \sum_{r=1}^{n} \sum_{k=r}^{n} \dfrac{1}{(k+1)} \left[\dfrac{1}{(k-r+1)} + \dfrac{1}{r}\right]

=k=1nr=1k1(k+1)[1(kr+1)+1r] = \sum_{k=1}^{n} \sum_{r=1}^{k} \dfrac{1}{(k+1)} \left[\dfrac{1}{(k-r+1)} + \dfrac{1}{r}\right]

=2k=1nHkk+1 = 2\sum_{k=1}^{n} \dfrac{H_{k}}{k+1}

Substitute k+1k k+1 \mapsto k

    S=2k=2n+1Hk1k\implies \text{S} = 2\sum_{k=2}^{n+1} \dfrac{H_{k-1}}{k}

=2k=2n+1Hk11kk = 2\sum_{k=2}^{n+1} \dfrac{H_{k-1} - \dfrac{1}{k}}{k}

=2k=1n+1Hkk2Hn+1(2) = 2\sum_{k=1}^{n+1} \dfrac{H_{k}}{k} - 2H_{n+1}^{(2)}

Since k=1n+1Hkk=12(Hn+12+Hn+1(2))\displaystyle \sum_{k=1}^{n+1} \dfrac{H_{k}}{k} = \dfrac{1}{2} (H_{n+1}^2 + H_{n+1}^{(2)}) (I have proved it here), we have,

S=(Hn+1)2Hn+1(2)\text{S} = (H_{n+1})^2 - H_{n+1}^{(2)} \quad \square

Ishan Singh - 4 years, 11 months ago

Problem 24:

Prove that k1F2kHk1(2)k2(2kk)=2π43755\displaystyle \sum_{k \geq 1} \frac{F_{2k}H_{k-1}^{(2)}}{k^2\binom{2k}{k}}=\frac{2\pi^4}{375\sqrt5}

Fn Fibonacci number F_n - \text{ Fibonacci number }

This problem has been solved by Aditya Kumar.

Aman Rajput - 4 years, 11 months ago

Log in to reply

Solution of Problem 24:

Lemma 1: x2=12n=1[22nn2(2nn)sin2nx]{ x }^{ 2 }=\frac { 1 }{ 2 } \sum _{ n=1 }^{ \infty }{ \left[ \frac { { 2 }^{ 2n } }{ { n }^{ 2 } \dbinom{2n}{n} } \sin ^{ 2n }{ x } \right] }

Proof:

Consider: fn(x)=sin2nx(2n)!{ f }_{ n }\left( x \right) =\frac { \sin ^{ 2n }{ x } }{ \left( 2n \right) ! } .

Then we get: f"n(x)=fn1(x)(2n)2fn(x)\\ { f" }_{ n }\left( x \right) ={ f }_{ n-1 }\left( x \right) -{ \left( 2n \right) }^{ 2 }{ f }_{ n }\left( x \right)

We can write x2=n=0anfn(x)\displaystyle { x }^{ 2 }=\sum _{ n=0 }^{ \infty }{ { a }_{ n }{ f }_{ n }\left( x \right) } for some function ana_n.

On differentiating both sides twice:

2=n=1an(fn1(x)(2n)2fn(x))=n=0an+1fn(x)n=1an(2n)2fn(x)2=\sum _{ n=1 }^{ \infty }{ { a }_{ n }\left( { f }_{ n-1 }\left( x \right) -{ \left( 2n \right) }^{ 2 }{ f }_{ n }\left( x \right) \right) } =\sum _{ n=0 }^{ \infty }{ { a }_{ n+1 }{ f }_{ n }\left( x \right) } -\sum _{ n=1 }^{ \infty }{ { a }_{ n }{ \left( 2n \right) }^{ 2 }{ f }_{ n }\left( x \right) }

From this relation a1=2a_1=2 and an+1=an(2n)2a_{n+1}=a_n{(2n)}^{2}

Thus, an2=k=1n1ak+1ak=22n2(n1)!2\displaystyle \frac { { a }_{ n } }{ 2 } =\prod _{ k=1 }^{ n-1 }{ \frac { { a }_{ k+1 } }{ { a }_{ k } } } ={ 2 }^{ 2n-2 }{ \left( n-1 \right) ! }^{ 2 }.

Hence, we get : x2=12n=1[22nn2(2nn)sin2nx]\displaystyle { x }^{ 2 }=\frac { 1 }{ 2 } \sum _{ n=1 }^{ \infty }{ \left[ \frac { { 2 }^{ 2n } }{ { n }^{ 2 } \dbinom{2n}{n} } \sin ^{ 2n }{ x } \right] }

Lemma 2: x4=12n=1[22nsin2nxHn1(2)n2(2nn)]{ x }^{ 4 }=\frac { 1 }{ 2 } \sum _{ n=1 }^{ \infty }{ \left[ \frac { { 2 }^{ 2n }\sin ^{ 2n }{ x } { H }_{ n-1 }^{ (2) } }{ { n }^{ 2 }\dbinom{2n}{n} } \right] }

Proof:

From Lemma 1, we can write: x2=12n=1[22nn2(2nn)sin2nx]=12n=1bn2fn(x)\displaystyle { x }^{ 2 }=\frac { 1 }{ 2 } \sum _{ n=1 }^{ \infty }{ \left[ \frac { { 2 }^{ 2n } }{ { n }^{ 2 } \dbinom{2n}{n} } \sin ^{ 2n }{ x } \right] }=\frac { 1 }{ 2 } \sum _{ n=1 }^{ \infty }{ { b }_{ n }^{ 2 }{ f }_{ n }\left( x \right) }

Here, bn=2n(n1)!b_n={ 2 }^{ n }(n-1)! and bn+1=2nbnb_{n+1}=2nb_n.

We can write: x4=12n=1anbn2fn(x)\displaystyle { x }^{ 4 }=\frac { 1 }{ 2 } \sum _{ n=1 }^{ \infty }{ { a }_{ n }{ b }_{ n }^{ 2 }{ f }_{ n }\left( x \right) } for some function ana_n.

On differentiating both sides twice:

12x2=12n=0an+1bn+12fn(x)12n=1anbn2(2n)2fn(x)12{ x }^{ 2 }=\frac { 1 }{ 2 } \sum _{ n=0 }^{ \infty }{ { a }_{ n+1 }{ b }_{ n+1 }^{ 2 }{ f }_{ n }\left( x \right) } -\frac { 1 }{ 2 } \sum _{ n=1 }^{ \infty }{ { a }_{ n }{ b }_{ n }^{ 2 }{ \left( 2n \right) }^{ 2 }{ f }_{ n }\left( x \right) }

Thus, we get: (an+1an)bn2(2n)2=12bn2({ a }_{ n+1 }-{ a }_{ n }){ b }_{ n }^{ 2 }{ \left( 2n \right) }^{ 2 }=12{ b }_{ n }^{ 2 }

From here we get: (an+1an)=12(2n)2({ a }_{ n+1 }-{ a }_{ n })=\frac { 12 }{ { \left( 2n \right) }^{ 2 } } .

Hence, an=Hn1(2){ a }_{ n }={ H }_{ n-1 }^{ (2) }.

Therefore, we get: x4=12n=1[22nsin2nxHn1(2)n2(2nn)]\displaystyle { x }^{ 4 }=\frac { 1 }{ 2 } \sum _{ n=1 }^{ \infty }{ \left[ \frac { { 2 }^{ 2n }\sin ^{ 2n }{ x } { H }_{ n-1 }^{ (2) } }{ { n }^{ 2 }\dbinom{2n}{n} } \right] }.

From lemma 2, we get: (sin1x)4=32n=122nHn1(2)n2(2nn)x2n ...(A)\displaystyle (\sin^{-1} x)^4 = \frac32 \sum_{n=1}^\infty \frac{2^{2n} H_{n-1}^{(2)}}{n^2 \binom{2n}{n}} \,x^{2n} \ \quad ...(A)

Now, in the problem, we use: F2k=φ2k1φ2k5{ F }_{ 2k }=\frac { { \varphi }^{ 2k }-\frac { 1 }{ { \varphi }^{ 2k } } }{ \sqrt { 5 } }

On substituting and using AA, we get: k1F2kHk1(2)k2(2kk)=2π43755\boxed{\displaystyle \sum_{k \geq 1} \frac{F_{2k}H_{k-1}^{(2)}}{k^2\binom{2k}{k}}=\frac{2\pi^4}{375\sqrt5}}

Aditya Kumar - 4 years, 11 months ago

Log in to reply

(+1) Nicely explained! I solved using Beta Functions and Integration.

Ishan Singh - 4 years, 11 months ago

Problem 20 :

Evaluate im=1nim1=1imi2=1i3i1=1i21\sum_{i_m=1}^n \sum_{i_{m-1}=1}^{i_m} \cdots \sum_{i_2=1}^{i_3} \sum_{i_1=1}^{i_2} 1

This problem has been solved by Aditya Sharma and Ameya Daigavane.

A Former Brilliant Member - 4 years, 11 months ago

Log in to reply

Although @Aditya Sharma has solved the question - I thought of an approach that might be considered more "illuminating" than induction.

Let's think of the i1,i2,...im i_1, i_2,...i_m as numbers on a register - each ik i_k is one number from 11 to nn. Initially, the register reads m m ones, because all the iki_k are 11.
For each unique number on the register, we add one to our count.

Now, we turn the first number (imi_m) to 22.
The register reads 22 followed by m1m - 1 11s.
Note that a digit to the right of another digit can never be greater than it, because ijiki_j \geq i_kif jkj \geq k.

So, the number on our register is an mm digit number whose digits are non-decreasing when read from left to right.
We can keep turning the register and counting how many unique numbers we get, until we reach mm digits with value nn. After this, our register stops working.

It is easy to see that every single number satisfying the non-decreasing digit condition is encountered, and only once.

So our question now becomes:
What is the number of tuples i1,i2,...im i_1, i_2,...i_m satisfying,

1i1i2imn 1 \leq i_1 \leq i_2 \leq \ldots \leq i_m \leq n

Because for each one of these tuples, we have a number on our register.

If we write the numbers in a row, i1i2im i_1 i_2 \ldots i_m and divide them into nn blocks, such that the numbers enclosed in the nthn^{th} block get value n n , then we have found one such possibility. We need n1n - 1 bars to separate the numbers into nn blocks.
For example, if m=4,n=4m = 4, n = 4, we can divide the numbers as, i1i2i3i4 i_1 | | i_2 i_3 | i_4 Here 0 i1=1,i2=i3=3,i4=4 i_1 = 1, i_2 = i_3 = 3, i_4 = 4

Clearly, the number of ways we can separate these numbers, is the number of tuples we need, because numbers with higher indices are always given higher (or equal) values.

So, we can finish with stars-and-bars, as we have mm numbers, and n1n -1 separators, so the total number of separations/tuples is: (m+n1n1) \binom {m + n - 1}{n - 1} which is the same answer as above.

Ameya Daigavane - 4 years, 11 months ago

Let S=im=1nim1=1imi2=1i3i1=1i21S =\sum_{i_m=1}^n \sum_{i_{m-1}=1}^{i_m} \cdots \sum_{i_2=1}^{i_3} \sum_{i_1=1}^{i_2} 1

S=im=1nim1=1imi2=1i3(i21)\displaystyle S=\sum_{i_m=1}^n \sum_{i_{m-1}=1}^{i_m} \cdots \sum_{i_2=1}^{i_3} \binom{i_2}{1}

S=im=1nim1=1imi3=1i4(i3+12)\displaystyle S = \sum_{i_m=1}^n \sum_{i_{m-1}=1}^{i_m} \cdots \sum_{i_3=1}^{i_4} \binom{i_3+1}{2}

S=im=1nim1=1imi4=1i5(i4+23)\displaystyle S = \sum_{i_m=1}^n \sum_{i_{m-1}=1}^{i_m} \cdots \sum_{i_4=1}^{i_5} \binom{i_4+2}{3}

It's obvious from induction that S=im=1n(im+m2m1)=nm(m+n1n)\displaystyle S=\sum_{i_m=1}^n \binom{i_m+m-2}{m-1} = \frac{n}{m}\binom{m+n-1}{n}

Aditya Narayan Sharma - 4 years, 11 months ago

Problem 21:

Prove that: n=0(2nn)5n=5\large \displaystyle \sum_{n=0}^{\infty} \binom{2n}{n} 5^{-n} = \sqrt{5}

This problem has been solved by Aditya Kumar.

Aditya Narayan Sharma - 4 years, 11 months ago

Log in to reply

Solution to Problem 21:

We use the well known generating function of Central Binomial coefficient:

114x=n=0(2n)!(n!)2xn\frac{1}{\sqrt{1 - 4x}} = \sum_{n=0}^\infty \frac{(2n)!}{(n!)^2}x^n

Here substitute x=15x=\frac{1}{5}.

Hence, n=0(2nn)5n=5\large \boxed{\displaystyle \sum_{n=0}^{\infty} \binom{2n}{n} 5^{-n} = \sqrt{5}}

Aditya Kumar - 4 years, 11 months ago

Log in to reply

Proof of that generating function?Thanks.

Harsh Shrivastava - 4 years, 11 months ago

Log in to reply

@Harsh Shrivastava There are many ways but I learnt it here.

Aditya Kumar - 4 years, 11 months ago

Problem 29:

Prove:

m=199sin(17mπ100)sin(39mπ100)1+cos(mπ100)=1037\displaystyle \sum\limits_{m=1}^{99}{\frac{\sin{\left(\frac{17 m \pi}{100}\right)} \sin{\left(\frac{39 m \pi}{100}\right)}}{1+\cos{\left( \frac{m\pi}{100} \right) }}}=1037

Due to time constraint, I have decided to post the solution.

Jack Lam - 4 years, 11 months ago

Log in to reply

Define T(k)=m=1n1cosmkπn1+cosmπn \displaystyle T(k) = \sum_{m=1}^{n-1} \frac{\cos{\frac{mk\pi}{n}}}{1+\cos{\frac{m\pi}{n}}}

Observing that cos(k+1)θ+2coskθ+cos(k1)θ=2coskθ(1+cosθ)\cos{(k+1)\theta} + 2\cos{k\theta} + \cos{(k-1)\theta} = 2\cos{k\theta}(1+\cos{\theta})

The following recurrence relation is obtained:

T(k+1)+2T(k)+T(k1)=2m=1n1cosmkπn=1+coskπ+sinkπtankπ2n\displaystyle T(k+1) + 2T(k) + T(k-1) = 2 \sum_{m=1}^{n-1} \cos{\frac{mk\pi}{n}} = 1+\cos{k\pi} + \sin{k\pi} \tan{\frac{k\pi}{2n}}

Note that an implicit restriction of knZ k \notin n \mathbb{Z} is forced because the GP formula is invalid when the common ratio is ±1\pm 1.

Evaluate the recurrence relation at k+1k+1 and add together to yield

T(k+2)+3T(k+1)+3T(k)+T(k1)=2 T(k+2) + 3T(k+1) + 3T(k) + T(k-1) = 2

Evaluate this recurrence relation at kk and take the difference to homogenise the recurrence relation:

T(k+2)+2T(k+1)2T(k1)T(k2)=0 T(k+2) + 2T(k+1) - 2T(k-1) - T(k-2) = 0

The characteristic roots are 11 and 1-1 with multiplicity 33.

The general solution to the recurrence relation is thus

(C1k2+C2k+C3)(1)k+C4 (C_1 k^2 + C_2 k+C_3)(-1)^k + C_4

Returning to the original problem, define S(a,b,n)=m=1n1sinamπnsinbmπn1+cosmπn\displaystyle S(a,b,n) = \sum_{m=1}^{n-1} \frac{\sin{\frac{am\pi}{n}}\sin{\frac{bm\pi}{n}}}{1+\cos{\frac{m\pi}{n}}} where 0ban 0 \le b \le a \le n

We find that 2S(a,b,n)=T(ab)T(a+b)2S(a,b,n) = T(a-b)-T(a+b)

Note that (1)a+b(1)ab (-1)^{a+b} \equiv (-1)^{a-b}

Substituting T(k)T(k) into the above and expanding, we obtain

S(a,b,n)=(2C1a+C2)b(1)a+b+1=(1)a+bC1^b(C2^a)\displaystyle S(a,b,n) = (2C_1 a + C_2)b(-1)^{a+b+1} = (-1)^{a+b}\hat{C_1}b(\hat{C_2}-a)

A short computation yields S(1,1,n)=n1S(1,1,n) = n-1

From S(n,b,n)=0&S(1,1,n)>0 S(n,b,n) = 0 \, \& \, S(1,1,n)>0, C2^=n\hat{C_2} = n

From S(1,1,n)=n1S(1,1,n) = n-1, C1^=1\hat{C_1} = 1

Therefore, we conclude S(a,b,n)=(1)a+bb(na) S(a,b,n) = (-1)^{a+b} b(n-a)

Substituting b=17,a=39,n=100b=17, a=39, n=100, and the result which was to be shown follows.

Jack Lam - 4 years, 11 months ago

Log in to reply

Nice solution. I missed the 48 hour time.

Aditya Kumar - 4 years, 11 months ago

This was the best solution!

A Former Brilliant Member - 4 years, 11 months ago

Problem 23 :\text{Problem 23 :}

Prove that : n=(1)n(n+a)4=π46cos(πa)sin2(πa)(6csc2(πa)1)\displaystyle \sum_{n=-\infty}^{\infty} \frac{(-1)^n}{(n+a)^4} = \frac{\pi^4}{6} \frac{\cos(\pi a)}{\sin^2 (\pi a)} (6\csc^2 (\pi a) -1) for aRa \in \mathbb{R} and aa is not an integer.

This problem has been solved by Aman Rajput.

Aditya Narayan Sharma - 4 years, 11 months ago

Log in to reply

n(1)n(n+a)4=(1)442(k01(k+1a2)4+1(k+a2)41(k1a2)41(k1+a2)4)\displaystyle \sum_{n\geq -\infty} \frac{(-1)^n}{(n+a)^4}=\frac{(-1)^4}{4^2}\left( \sum_{k \geq 0}\frac{1}{(k+1-\frac{a}2)^4}+\frac{1}{(k+\frac{a}2)^4}-\frac{1}{(k-\frac{1-a}2)^4}-\frac{1}{(k-\frac{1+a}2)^4} \right)

=116(16(ψ(3)(1a2)+ψ(3)(a2)ψ(3)(1a2)ψ(3)(1+a2)))\frac{1}{16} (\frac16(\psi^{(3)}(1-\frac{a}2)+\psi^{(3)}(\frac{a}2)-\psi^{(3)}(\frac{1-a}2)-\psi^{(3)}(\frac{1+a}2)))

Using reflection formula and On a little bit solving you get

π46cos(πa)sin2(πa)(6csc2(πa)1)\frac{\pi^4}{6}\frac{cos(\pi a)}{\sin^2(\pi a)}(6\csc^2(\pi a)-1) Sorry for short solution

Aman Rajput - 4 years, 11 months ago

Log in to reply

Yup its ok. Nice solution .

Aditya Narayan Sharma - 4 years, 11 months ago

Alternative :\text{Alternative :}

Consider the function f(z)=1(z+a)4\displaystyle f(z)=\frac{1}{(z+a)^4} which analytic and has poles at the point z=az=-a of order 4.

Considering the square contour positively oriented having vertices ±(1+i) \pm (1+i) and by residue lemma we derive ,

Res(a)=16limzad3dz3(πcsc(πz)) \displaystyle Res(-a) = \frac{1}{6} \lim_{z\to -a} \frac{d^3}{dz^3}(π\csc(πz))

Finally using alternate summation theorem ,

n=(1)n(n+a)4=16limzad3dz3(πcsc(πz))=π46cos(πa)sin2(πa)(6csc2(πa)1)\displaystyle \sum_{n=-\infty}^{\infty} \frac{(-1)^n}{(n+a)^4} =- \frac{1}{6} \lim_{z\to -a} \frac{d^3}{dz^3}(π\csc(πz)) = \frac{\pi^4}{6} \frac{\cos(\pi a)}{\sin^2 (\pi a)} (6\csc^2 (\pi a) -1)

which gives the result as desired.

Aditya Narayan Sharma - 4 years, 11 months ago

Log in to reply

firstly , i thought it can be solved using laurent series... but i thought we dont go to the complex analysis . :) nice

Aman Rajput - 4 years, 11 months ago

Log in to reply

@Aman Rajput Its nice to have both real and complex solutions of a problem for enlightenment as we now have for this one

Aditya Narayan Sharma - 4 years, 11 months ago

Problem 25:

Evaluate:

r=1nHr2\sum _{ r=1 }^{ n }{ { H }_{ r }^{ 2 } }

This problem has been solved by Aman Rajput.

Aditya Kumar - 4 years, 11 months ago

Log in to reply

Using Summation by parts k=1nHk2=Hn((n+1)Hnn)1kn1(k+1)Hkkk+1\displaystyle \sum _{ k=1 }^{ n }{ { H }_{ k }^{ 2 } } = H_n((n+1)H_n-n)-\sum_{1\leq k \leq n-1}\frac{(k+1)H_k-k}{k+1}

=(n+1)Hn2nHnnHn1+n1+nHn=\displaystyle (n+1)H_n^2-nH_n-nH_{n-1}+n-1+n-H_n

Aman Rajput - 4 years, 11 months ago

Problem 26: Prove : k1Hk(4)Hk12(4)2k+1=2ζ(2)ζ(3)(15ln214)ζ(4)\displaystyle \sum_{k \geq 1}\frac{H_k^{(4)}-H_{k-\frac12}^{(4)}}{2k+1}=2\zeta(2)\zeta(3)-(15\ln 2-14)\zeta(4)

This problem has been solved by Aditya Kumar.

Aman Rajput - 4 years, 11 months ago

Log in to reply

Partial fractions used here: 32n324n2+8n1(2n+1)(n)4(2n1)4=1n4+2n34n212n1152n+1+2(2n1)24(2n1)3+8(2n1)4+8n\frac { { 32n }^{ 3 }-{ 24n }^{ 2 }+8n-1 }{ \left( 2n+1 \right) { \left( n \right) }^{ 4 }{ \left( 2n-1 \right) }^{ 4 } } =-\frac { 1 }{ { n }^{ 4 } } +\frac { 2 }{ { n }^{ 3 } } -\frac { 4 }{ { n }^{ 2 } } -\frac { 1 }{ { 2n }-1 } -\frac { 15 }{ 2{ n }+1 } +\frac { 2 }{ { \left( 2n-1 \right) }^{ 2 } } -\frac { 4 }{ { \left( 2n-1 \right) }^{ 3 } } +\frac { 8 }{ { \left( 2n-1 \right) }^{ 4 } } +\frac { 8 }{ { n } }

Formula (1) used: Hqp(m)=ζ(m)pmn=11(q+pn)m{ H }_{ \frac { q }{ p } }^{ (m) }=\zeta \left( m \right) -{ p }^{ m }\sum _{ n=1 }^{ \infty }{ \frac { 1 }{ { \left( q+pn \right) }^{ m } } }

Formula (2) used: k=1p12k+1=12(Ψ(p+32)Ψ(32))\sum _{ k=1 }^{ p }{ \frac { 1 }{ 2k+1 } } =\frac { 1 }{ 2 } \left( \Psi \left( p+\frac { 3 }{ 2 } \right) -\Psi \left( \frac { 3 }{ 2 } \right) \right)

n=1[Hn(4)H2n12(m)2n+1]=n=1[16k=11(2n+2k1)4k=11(n+k)42n+1]\sum _{ n=1 }^{ \infty }{ \left[ \frac { { H }_{ n }^{ (4) }-{ H }_{ \frac { 2n-1 }{ 2 } }^{ (m) } }{ 2n+1 } \right] } =\sum _{ n=1 }^{ \infty }{ \left[ \frac { 16\sum _{ k=1 }^{ \infty }{ \frac { 1 }{ { \left( 2n+2k-1 \right) }^{ 4 } } } -\sum _{ k=1 }^{ \infty }{ \frac { 1 }{ { \left( n+k \right) }^{ 4 } } } }{ 2n+1 } \right] }

=n=1k=1[16(n+k)4(2n+2k1)4(2n+1)(n+k)4(2n+2k1)4]=\sum _{ n=1 }^{ \infty }{ \sum _{ k=1 }^{ \infty }{ \left[ \frac { 16{ \left( n+k \right) }^{ 4 }-{ \left( 2n+2k-1 \right) }^{ 4 } }{ \left( 2n+1 \right) { \left( n+k \right) }^{ 4 }{ \left( 2n+2k-1 \right) }^{ 4 } } \right] } }

=n=1k=n+1[16(k)4(2k1)4(2n+1)(k)4(2k1)4]=\sum _{ n=1 }^{ \infty }{ \sum _{ k=n+1 }^{ \infty }{ \left[ \frac { 16{ \left( k \right) }^{ 4 }-{ \left( 2k-1 \right) }^{ 4 } }{ \left( 2n+1 \right) { \left( k \right) }^{ 4 }{ \left( 2k-1 \right) }^{ 4 } } \right] } }

=n=1k=n+1[32(k)324(k)2+8k1(2n+1)(k)4(2k1)4]=\sum _{ n=1 }^{ \infty }{ \sum _{ k=n+1 }^{ \infty }{ \left[ \frac { 32{ \left( k \right) }^{ 3 }-{ 24\left( k \right) }^{ 2 }+8k-1 }{ \left( 2n+1 \right) { \left( k \right) }^{ 4 }{ \left( 2k-1 \right) }^{ 4 } } \right] } }

=n=1[k=n[32(k)324(k)2+8k1(2n+1)(k)4(2k1)4]32(n)324(n)2+8n1(2n+1)(n)4(2n1)4]=\sum _{ n=1 }^{ \infty }{ \left[ \sum _{ k=n }^{ \infty }{ \left[ \frac { 32{ \left( k \right) }^{ 3 }-{ 24\left( k \right) }^{ 2 }+8k-1 }{ \left( 2n+1 \right) { \left( k \right) }^{ 4 }{ \left( 2k-1 \right) }^{ 4 } } \right] } -\frac { 32{ \left( n \right) }^{ 3 }-{ 24\left( n \right) }^{ 2 }+8n-1 }{ \left( 2n+1 \right) { \left( n \right) }^{ 4 }{ \left( 2n-1 \right) }^{ 4 } } \right] }

Now, we change the order of the double sum:

=k=1n=1k[32(k)324(k)2+8k1(2n+1)(k)4(2k1)4]n=1[32(n)324(n)2+8n1(2n+1)(n)4(2n1)4]=\sum _{ k=1 }^{ \infty }{ \sum _{ n=1 }^{ k }{ \left[ \frac { 32{ \left( k \right) }^{ 3 }-{ 24\left( k \right) }^{ 2 }+8k-1 }{ \left( 2n+1 \right) { \left( k \right) }^{ 4 }{ \left( 2k-1 \right) }^{ 4 } } \right] } } -\sum _{ n=1 }^{ \infty }{ \left[ \frac { 32{ \left( n \right) }^{ 3 }-{ 24\left( n \right) }^{ 2 }+8n-1 }{ \left( 2n+1 \right) { \left( n \right) }^{ 4 }{ \left( 2n-1 \right) }^{ 4 } } \right] }

Now using the 3 formulas I have provided above and evaluating each summation (which is basic wrt this contest), on simplification gives:

k1Hk(4)Hk12(4)2k+1=2ζ(2)ζ(3)(15ln214)ζ(4)\displaystyle \sum_{k \geq 1}\frac{H_k^{(4)}-H_{k-\frac12}^{(4)}}{2k+1}=2\zeta(2)\zeta(3)-(15\ln 2-14)\zeta(4)

The simplification is left as an exercise to the readers.

Aditya Kumar - 4 years, 11 months ago

Log in to reply

justify your last double summation. partial fraction is justified. @Aditya Kumar

Aman Rajput - 4 years, 11 months ago

Log in to reply

@Aman Rajput See formula 2 for that.

Aditya Kumar - 4 years, 11 months ago

Problem 19 :

Prove That

r=1nHr(2)=(n+1)Hn(2)Hn \sum_{r=1}^{n} H_{r}^{(2)} = (n+1)H_{n}^{(2)} - H_{n}

Notation : Hn(m)=k=1n1km\displaystyle H_{n}^{(m)} = \sum_{k=1}^{n} \dfrac{1}{k^m} denotes the Generalized Harmonic Number.

This problem has been solved by Deeparaj Bhat.

Ishan Singh - 4 years, 11 months ago

Log in to reply

r=1nHr(2)=r=0n1nr(1+r)2=r=0n1n+1(1+r)211+r=(n+1)Hn(2)HnQ.E.D. \begin{aligned} \sum_{r=1}^n H_r^{(2)} &= \sum_{r=0}^{n-1} \frac{n-r}{(1+r)^2} \\&= \sum_{r=0}^{n-1} \frac{n+1}{(1+r)^2} - \frac{1}{1+r} \\&= (n+1)H_n^{(2)} - H_n \\& Q. E. D. \end{aligned}

A Former Brilliant Member - 4 years, 11 months ago

Problem 30 :

Evaluate:

n=13n(2nn)\displaystyle \sum_{n=1}^{\infty} \frac{3^n}{\binom{2n}{n}}

Jack Lam - 4 years, 11 months ago

Log in to reply

01n1(2n+1)(3x3x2)ndx\displaystyle \int\limits_{0}^{1}\sum_{n\geq 1} (2n+1)(3x-3x^2)^n dx

901x42x3+2x2x(3x23x+1)2dx\displaystyle -9\int\limits_{0}^{1} \frac{x^4-2x^3+2x^2-x}{(3x^2-3x+1)^2} dx

927(943π)\frac{-9}{27}(-9-4\sqrt3\pi)

13(9+43π)\boxed{\frac13(9+4\sqrt3\pi)}

Aman Rajput - 4 years, 11 months ago

Note that,

f(x)=2arcsin(x2)=n=1xnn2(2nn) f(x) = 2 \arcsin \left( \dfrac{x}{2} \right) = \sum_{n=1}^{\infty} \dfrac{x^n}{n^2 \dbinom{2n}{n}}

The series is xddx(xddxf(x)) \displaystyle x \dfrac{\text{d}}{\text{d}x} \left(x \dfrac{\text{d}}{{\text{d}x}} f(x) \right) at x=3x=3

=3+4π3 = 3+ \dfrac{4 \pi}{\sqrt{3}}

Ishan Singh - 4 years, 11 months ago

Problem 28: Prove:

n1(1)n+1Hnn2(n+44)=58ζ(3)2524ζ(2)+163ln2(2)283ln2+727144\displaystyle \sum_{n \geq 1} \frac{(-1)^{n+1}H_n}{n^2 \binom{n+4}{4}}=\frac58\zeta(3)-\frac{25}{24}\zeta(2)+\frac{16}{3}\ln^2(2)-\frac{28}{3}\ln 2 +\frac{727}{144}

Aman Rajput - 4 years, 11 months ago

Log in to reply

Starting with n=1Hnxnn=0xln(1u)u(1u)du=Li2(x)+12ln2(1x) \sum_{n=1}^\infty \frac{H_n x^n}{n} = -\int_0^x \frac{\ln(1-u)}{u(1-u)}\,du = \mathrm{Li}_2(x) + \tfrac{1}{2}\ln^2(1-x) we have f(x)=n=1(1)n+1Hnxnn=Li2(x)12ln2(1+x)f(x) = \sum_{n=1}^\infty \frac{(-1)^{n+1}H_n x^n}{n} = -\mathrm{Li}_2(-x) -\tfrac12\ln^2(1+x) Then, on reordering a multiple integral, S=n=1(1)n+1Hnn2(n+44)  =  24n=1(1)n+1Hnn2(n+1)(n+2)(n+3)(n+4)=2401dp0pdq0qdr0rds0sf(t)tdt=01(1t)4f(t)tdt=01(1t)4t{Li2(t)+12ln2(1+t)}dt\begin{array}{rcl} S &=& \displaystyle\sum_{n=1}^\infty \frac{(-1)^{n+1}H_n}{n^2 \binom{n+4}{4}} \; = \; 24\sum_{ n=1}^{\infty} \frac{(-1)^{n+1}H_n}{n^2(n+1)(n+2)(n+3)(n+4)}\\ &=& \displaystyle24\int_0^1dp \int_0^ p dq\int_0^q dr \int_0^r ds\int_0^s \frac{f(t)}{t}\, dt \\ &=& \displaystyle\int_0^1 \frac{(1-t)^4 f(t)}{t}\,dt \\ &=&\displaystyle -\int_0^1 \frac{(1-t)^4}{t}\big\{\mathrm{Li}_2(-t)+\tfrac12\ln^2(1+t)\big\}\, dt \end{array} These integral are easy -even WA can do them -and the answer is a simple calculation.

Let Jack post the next question.

Mark Hennings - 4 years, 11 months ago

Log in to reply

This is called solution . Very nice sir ! Jusified clearly

Aman Rajput - 4 years, 11 months ago

So far I've managed to arrive at a sum of sums by the method of Partial Fractions.

In the hopes it will help anyone, here is the decomposition.

1n2(n+44)1n22512n+4n+13n+2+43(n+3)14(n+4) \frac{1}{n^2 \binom{n+4}{4}} \equiv \frac{1}{n^2} - \frac{25}{12n} + \frac{4}{n+1} - \frac{3}{n+2} + \frac{4}{3(n+3)} - \frac{1}{4(n+4)}

Jack Lam - 4 years, 11 months ago

Log in to reply

You can proceed here by using the generating function.

Aditya Kumar - 4 years, 11 months ago

Proceeding from the decomposition of the denominator, we break the sum into six parts.

(1)n+1Hnn2(n+44)(1)nHnn2+2512(1)nHnn+4(1)n+1Hnn+1+3(1)n+2Hnn+2+43(1)n+3Hnn+3+(1)n+44(n+4) \frac{(-1)^{n+1}H_n}{n^2 \binom{n+4}{4}} \equiv -\frac{(-1)^n H_n}{n^2} + \frac{25}{12} \cdot \frac{(-1)^n H_n}{n} + \frac{4(-1)^{n+1} H_n}{n+1} + \frac{3(-1)^{n+2}H_n}{n+2} + \frac{4}{3} \cdot \frac{(-1)^{n+3} H_n}{n+3} + \frac{(-1)^{n+4}}{4(n+4)}

Note the powers have been adjusted to match the denominator.

By using the Generating Function for the Harmonic Numbers and swapping the order of integration and summation, we obtain lots of logarithms, and a few Dilogarithms and Trilogarithms.

Use the Polylogarithm identities found on here: http://mathworld.wolfram.com/Trilogarithm.html and here: http://mathworld.wolfram.com/Dilogarithm.html to simplify everything.

Jack Lam - 4 years, 11 months ago

Log in to reply

yes. That was one method. Posting all the steps is indeed tedious.

Aditya Kumar - 4 years, 11 months ago

Log in to reply

@Aditya Kumar I look forward to an elegant solution :)

Jack Lam - 4 years, 11 months ago

its better if you paste screenshots of wolframalpha.. that would help here

Aman Rajput - 4 years, 11 months ago

Log in to reply

@Aman Rajput

Jack Lam - 4 years, 11 months ago

Log in to reply

@Jack Lam Hahaha ... 😂😂😂😂😂😂😂

Aman Rajput - 4 years, 11 months ago

Log in to reply

@Aman Rajput You asked for it, I presented it :)

Jack Lam - 4 years, 11 months ago

Log in to reply

@Jack Lam i was not asking these .. i was asking the solution to the problem.

Aman Rajput - 4 years, 11 months ago

Log in to reply

@Aman Rajput The solution I happen to have come up with is extremely tedious. One must use all of the above to evaluate the sum.

I do not know of a faster way to achieve the result.

Jack Lam - 4 years, 11 months ago

Log in to reply

@Jack Lam you should post it . how we can believe your solution is correct.

Aman Rajput - 4 years, 11 months ago

@Jack Lam i also can add a solution to your problem using wolframalpha and put a screenshot there. But that wont be good.

Aman Rajput - 4 years, 11 months ago

@Jack Lam i know that the result you have posted and the screenshot does not match at all. I know this cant be solved using that. If you know then prove that you are right and i am wrong . If you cant then delete your question within a day. you have one more day to justify your result

Aman Rajput - 4 years, 11 months ago

Bonus Problem: Evaluate :

n=(e12π)n2\displaystyle \sum_{n=-\infty}^{\infty}(-e^{-12\pi})^{n^2}


None has solved this question till 5 days

Aman Rajput - 4 years, 11 months ago

Log in to reply

...knock knock, is this comp over?

Hung Woei Neoh - 4 years, 10 months ago

Let k=π1/4Γ(3/4)k=\frac{\pi^{1/4}}{\Gamma(3/4)}

ϕ(e2nπ)=kr2,2n2n1/421/4rn,n\phi(-e^{-2n\pi})=\frac{kr_{2,2n^2}}{n^{1/4}2^{1/4}r_{n,n}}

Put n=6n=6 We have r6,6=31/81+3(1+3+233/4)1/3213/24r_{6,6}=\frac{3^{1/8}\sqrt{1+\sqrt3}(1+\sqrt3+\sqrt23^{3/4})^{1/3}}{2^{13/24}}

r2,72=η(6i)21/4η(12i)r_{2,72}=\frac{\eta(6i)}{2^{1/4}\eta(12i)}

solving this function, and putting back into equation we get

ϕ(e12π)=k219/4833/8(232+35/4+33/4)1/3(21)1/12(3+1)1/6(13+233/4)1/3\phi(-e^{-12\pi})=\frac{k2^{-19/48}3^{-3/8}(2-3\sqrt2+3^{5/4}+3^{3/4})^{1/3}}{(\sqrt2-1)^{1/12}(\sqrt3+1)^{1/6}(-1-\sqrt3+\sqrt23^{3/4})^{1/3}}

Aman Rajput - 4 years, 10 months ago

@Aman Rajput Sir, could you please suggest me some good books from where I can learn these type of summations from???

Aaghaz Mahajan - 2 years, 7 months ago

Log in to reply

Such kind of things are not in books. Go for research papers developed by great mathematicians from different universities. Sometimes it requires computers mathematical software like mathematica or maple to solve questions.

Aman Rajput - 2 years, 6 months ago

Log in to reply

@Aman Rajput Ohhh......I see........Well, I was already studying some books regarding integrals, and they turned out great!!! So, yeah, now I'll browse through papers too...........!!! Thanks..... :)

Aaghaz Mahajan - 2 years, 6 months ago
×

Problem Loading...

Note Loading...

Set Loading...